« first day (1516 days earlier)      last day (1093 days later) » 
06:00 - 15:0015:00 - 17:00

6:09 AM
@JohnRennie Hi!
 
@Wolgwang hi :-)
 
Are you free?
 
Yes
 
> A person is standing on a road by holding an umbrella at right angle to the horizontal in order to protect himself from rain falling at speed of 10m/s,and where as the velocity of wind is 0. Suddenly wind blows at a speed of 20m/s, towards 30degree south of west. Now by what angle the person has to turn the umbrella to protect himself from rain?
$$V_{Init.rain}=-10\hat k\\V_w=-10\sqrt3 \hat{i} -10\hat j\\V_{Finalrain}=-10\sqrt3 \hat{i} -10\hat j-10\hat k$$
What to do next?
 
You just need to find the angle of the vector along which the rain moves. Yes?
 
6:13 AM
Yes
 
I don't think towards 30degree south of west makes any difference.
 
Hmm?
 
The important point is that the wind is blowing horizontally at 20 m/s and the rain is falling vertically at 10 m/s
So the angle the rain makes with the horizontal is tan θ = 10/20
I can draw a diagram if it will help ...
 
Yes :-)
 
@Wolgwang the rain falls down through the air at 10 m/s, and the air is moving horizontally at 20 m/s. Yes?
 
6:22 AM
@JohnRennie Yes
 
So the rain travels along the red vector.
 
Yes
 
And you need to turn your umbrella so it points along the red vector.
 
Yes
 
Since you started with the umbrella pointing straight up you need to turn your umbrella by an angle of 90 - θ. Yes?
 
6:24 AM
But I also need to turn the umbrella on the z-axis.
 
Umbrellas are axially symmetric. An umbrella has a top and bottom, but not a front or back. So you do not need to rotate the umbrella about the z axis.
That's why towards 30degree south of west makes no difference.
 
:-o Didn't know that
 
Have you never seen an umbrella?
 
I have
 
Well if you are holding an umbrella straight up, i.e. along the z axis, then rotating the umbrella about this axis doesn't change it. Yes?
 
6:29 AM
What does rotating about the axis mean?
Changing vector's head in xy?
 
Here is a top down view of an umbrella I found on Google images:
So we are looking straight downwards and you are sheltering underneath the umbrella.
i.e. we are looking down the z axis. Yes?
 
Ok I got it
 
:-)
 
Thanks :-)
 
@JohnRennie Hii sir
I have quite a few questions ask.
Are you free rn?
Whenever you are. This is the Q
Q1 Now , average speed of particles = $\frac{\sqrt{8RT}}{\sqrt{ \pi*M}}$ where M is Molar mass of the gas.

Pressure by real gases : $\frac{RnT}{V-(nb)}$ - $\frac{n^2}{v^2}$-a . Here , a is strength of attraction between particles and b is volume occupied by mass of 1 mole particle.

So , My Q is that why don’t we consider in either of the formula the repulsion forces,collision between the particles ? Like , it is not mentioned anywhere in these equations.
Q2 is this Here. There are two gases He both.


Volume of above is 8L and below is 2L


Left side diagram is Q


We have founded out the pressure for both the gases on right


Then subtracted them


Now , what I got earlier was that the below piston will surely move up because we also say that the temperature is increasing gradually


It keeps on increasing


Coming back , P of gas below piston - P of gas above piston = weight of piston / Area of piston? This is what I didn’t get. Why the weight of piston ?
There are two Q here. We can with anyone first
I am here. Just ping me when we can discuss sir
 
6:43 AM
@SrijanM.T hi, yes I'm free.
The equation $v_{rms} = \frac{\sqrt{8RT}}{\sqrt{ \pi*M}}$ is for an ideal gas.
 
Yes sir
 
So in that case there are no interactions between the gas molecules.
If there are interactions then the equation becomes a lot more complicated.
 
Ohk.Accepted for ideal gas
@JohnRennie Right
What about real gas scenario
the equation I wrote second one
 
For an ideal gas $PV = nRT$. Yes?
 
Yes
 
6:46 AM
For a real gas the equation will be much more complicated, but we can make some approximations.
 
Ohk.
I wrote it sir. For real gas the equation
 
The volume is the volume that the gas molecules are free to move in, but if the gas molecules are not points (like in an ideal gas) then they take up some volume $b$.
 
@JohnRennie Yes sir
 
So the volume the gas molecules can move in is $V-b$. So we can change our equation to:
$$ P(V-b) = nRT$
 
Yes
 
6:48 AM
For some constant $b$ that depends on the size of the gas molecules.
 
Yes.
 
And now suppose there is an attractive force between the gas molecules.
 
Ohk.
Since they have mass. So , there is attraction
 
Since this pulls the gas molecules together it will reduce the pressure the gas molecules exert on the container they are in. Yes?
 
Yes
 
6:49 AM
So the pressure is going to be reduced a bit to $P-x$ where $x$ is some value.
 
Yes
 
It turns out that a good choice for $x$ is $a/V_m^2$ where $V_m$ is the molar volume.
 
Ohk.
 
And $V_m = V/n$ i.e. volume per mole, so that makes $x = an^2/V^2$
And our equation becomes:
 
Yes sir. Right
 
6:52 AM
$$ \left( P - \frac{an^2}{V^2} \right) (V - b ) = nRT $$
 
Yes sir right
 
But this is an approximation.
 
Ohk.Yes
 
Real gases would be far more complicated.
 
@JohnRennie K
 
6:53 AM
That approximation gives a reasonably good description for a real gas like nitrogen or oxygen.
 
We considered attraction forces I,e the value “a”
@JohnRennie K
 
And it's called the Van der Waal's equation of state.
 
But what about repulsion , collision forces for it. Since real gas particles have mass
@JohnRennie Yes sir, ok
 
@SrijanM.T yes, $a$ is a constant that describes the attraction. The repulsion is included in the constant $b$.
 
Sir , b is the amount of volume occupied by 1 mole particles right ?
 
6:55 AM
No, $b$ is approximately the volume occupied by a mole of gas molecules, but only approximately.
 
Ohk.
But sir , how does this b tell us about repulsion force ?
 
In practice we have to make measurements on a real gas and calculate the values of a and b from those measurements.
 
It talks about volume right
 
Repulsion means it's hard to squeeze two molecules together. Yes?
 
I am thinking repulsion as when two molecules collide with each other and then go away from each with more velocity
 
6:57 AM
If it wasn't hard to squeeze two molecules together they would never collide because they could get arbitrarily close to each other.
So collisions occur because of repulsive forces. Yes?
 
Yes
 
So:
2 mins ago, by John Rennie
Repulsion means it's hard to squeeze two molecules together. Yes?
 
Ohk. Got it
@JohnRennie I can do the maths after that.
Got this Q. Shall we move to 2nd one now ?
 
OK :-)
Q2 is this Here. There are two gases He both.
Volume of above is 8L and below is 2L
Left side diagram is Q
We have founded out the pressure for both the gases on right
Then subtracted them
Now , what I got earlier was that the below piston will surely move up because we also say that the temperature is increasing gradually
It keeps on increasing
Coming back , P of gas below piston - P of gas above piston = weight of piston / Area of piston? This is what I didn’t get. Why the weight of piston ?
https://i.stack.imgur.com/rQXwf.jpg
 
Yes sir
 
7:02 AM
 
Yes sir.
 
Give me a moment to read through the question.
 
Ohk. Sure
 
> Coming back , P of gas below piston - P of gas above piston = weight of piston / Area of piston? This is what I didn’t get. Why the weight of piston ?
Call the pressure above P₁ and the pressure below P₂
So if the area of the piston is A then the gas above it exerts a downwards force on the piston P₁A and the gas below exerts an upwards force P₂A. The net upwrds force on the piston due to the gas is Fup = (P₂ - P₁)A.
OK so far?
 
One sec
Yes sir right
 
7:08 AM
And the weight of the piston exerts a downwards force Fdown = mg. Yes?
 
Yes
 
So the net force on the piston is:
Fnet = Fup - Fdown = (P₂ - P₁)A - mg
 
OH yes
I got it
@JohnRennie Amazing
 
:-)
 
Sir , if you’re free. I have one more Q
 
7:09 AM
OK ... ?
 
More of problem solving
1 min. I’ll paste it
 
OK, how far did you get with this?
 
Two chambers having certain PVT. After stopper is removed , Q is to find Final pressure on both sides
@JohnRennie Very far. I will explain you the solution . Pls correct me
 
OK ...
 
There are two cases here. One is when T is constant even though pressure changes and other one is when T is not constant
Solution for case 1:
When the stopper is removed , our final conclusion is that pressure is same on both sides. Ok ?
 
7:14 AM
Yes
 
So , let us assume no of moles on left side = L and on RHS = R
 
Yes, that's how I would start.
 
Now , assuming that no of mole particles travel more RHS to LHS. Since particles flow from high pressure to low pressure. It is just an assumption that RHS has more particles but in end of our ans comes -ve , then we know that transition happening from LHS to RHS
Ok so far ?
 
Yes
 
@JohnRennie Feel good to know that
Now , we can say
R - x( x means the no of particles that went from RHS to LHS)
 
7:17 AM
I think you may be making this more complicated than it needs to be ...
 
and L+ x( means that those x particles have traveled to LHS)
@JohnRennie There are two methods I will solve it with
This one is lengthy. Not second one
 
Just bear with me till 2nd one. After that , I’ll discuss with you
 
R(L+x) T1/V1 = R(n_R+x)(T2)/V2
The R in bracket for 2nd one means moles
So , by doing this. We can calculate x and then in end , find P final
So , this is 1 method
Pls check if it is correct
 
7:20 AM
OK. Though you cannot assume the temperatures stay constant ...
 
@JohnRennie Yes. But here , it is given in Q to assume them to be cosnt
 
In case 2 when we do , I won’t assume that
Ok. Now , METHOD 2, not. CASE 2
Now , let us say n_L = 20 and n_R = 10
After solving , just keeping simple values for now
Ok so far ?
 
Yes
 
Now , I Cassie that molecules move from RHS to LHS. So , since the temp is constant. The molecules from RHS to LHS cool down by 300K
Due to temp difference
We can say it cools down by 2/3rd factor
Ok so far ?
 
7:25 AM
OK
By ²⁄₃ or by ¹⁄₃?
 
2/3
It becomes 600K on right
900*2/3
Since P= nRT/V
So T changes by 2/3rd
Ok so far ?
 
So , I can 10-X from RHS
But for LHS , Volume has decreased when they entered into LHS
Also , P=nRT/V
So , if T changes by 2/3rd factor , then also P changes by same factor
So , it will be 20+X(2/3)
Ok so far ?
It took me hours to think about this method . I hope it’s ok what I have done till now
There is little more
 
OK ...
 
So , now . We know P is directly proportional to V
That has change of 3rd factor
So , in final it becomes 20+X(2/3)(3)=10-X
I knew about this method till constant volume and temp. I thought to take not constant volume , so I thought of this then.
Ok so far? Done now sir
I hope this is the one you were thinking too
 
7:35 AM
I as thinking of the situation where the temperature equilibrates so it's equal on both sides, but if the question says to keep the temperatures unchanged that wouldn't apply.
 
@JohnRennie ohk.
Q says that even after changing moles , T is 600 and 900 K only
On LHS and rhs
 
I guess what I'd say is to use P = nRT/V
 
Hmm K. The only problem with that is that it can become lengthy
 
So start by calculating n₁ and n₂ before the tap is opened.
 
@JohnRennie Yes. That is right. In Q where values like R is there and then V is also difficult value. It can be lengthy to find nL
and nR
 
7:38 AM
And after the tap is opened the amount of gas will change to N₁ and N₂, but the total amount of gas has to stay the same so N₁ and N₂ = n₁ + n₂
And we know with the tap open P₁ = P₂
 
What is N?
Is it V?
 
Number of moles after the tap is opened.
 
N1 and n1 are same ?
 
I called it N to distinguish it from the number of moles before the tap is opened n
 
Ohk.
@JohnRennie Yes
Sir. Shall we assume T is not constant for this case. That is what I am finding difficult to solve
T1=T2 here. Unlike the 1st one
 
7:41 AM
So after the tap is open N₁RT₁/V₁ = N₂RT₂/V₂. Yes?
 
Yes. Any difference if T1=T2 or T1 is not equal to T2 ?
In the equation
Oh. I think T will get cut
 
This is always true, but if we allow the temperatures to change we no longer know the values of T₁ and T₂.
Just to be clear, what calculation are we trying to do? Constant temp or changing temp?
 
Yes. So , it should be T on both sides right. Just like we considered P final of both to be same
 
And if it's changing temp are we assuming the temp is the same on both sides?
 
@JohnRennie I want to do changing sir. I was confused about it a bit
@JohnRennie Yes
@JohnRennie I think so right ?
 
7:45 AM
If we allow the temp to equilibrate to be the same on both sides then it's easy because then we have P (V₁ + V₂) = (n₁ + n₂)RT. Yes?
 
Sir. Are we assuming both the chambers as one system ?
 
Yes. With the tap open it's one system because the gas molecules can move freely between each side.
 
Ohk. So , I was correct about T assumption
Like it will get cut on both sides
N₁RT/V₁ = N₂RT/V₂.
 
And we know the internal energy has not changed because no heat was added and no work was done.
 
@JohnRennie ohk. Right
Sir, I have one confusion. How is N1 AND N2 both n1+n2
 
7:49 AM
So that means n₁CT₁ + n₂CT₂ = (n₁ + n₂)CT.
 
One has to be n1-x and other n2+x right ?
 
@SrijanM.T forget about the uppercase N's. That was for a different calcualion.
46 secs ago, by John Rennie
So that means n₁CT₁ + n₂CT₂ = (n₁ + n₂)CT.
 
Ok. C is heat capacity ?
 
Yes
 
Ohk.
@JohnRennie So state 1 + state 2 = final state ?
 
7:50 AM
So T = (n₁T₁ + n₂T₂)/(n₁ + n₂)
 
@JohnRennie Didn’t get this one sir ?
 
@SrijanM.T I'm just saying the internal energy before the tap is opened is Uinit = n₁CT₁ + n₂CT₂ and after the tap is opened and the two sides have equilibrated Ufinal = (n₁ + n₂)CT where T is the final temperature. Yes?
 
Yes
 
And the internal energy is constant so Uinit = Ufinal. Yes?
 
I don’t know about internal energy actually. Sorry. I am yet to do thermo. My only Q Uinital = state 1 + state 2 means that both the states are considered as one system
So , final system is also kind of line one system
Because in the end, also the final system is same on both sides .
@JohnRennie So , I got it.
Sir @JohnRennie ? Are you there
 
7:58 AM
So Uinit = Ufinal and substituting for Uinit = n₁CT₁ + n₂CT₂ and Ufinal = (n₁ + n₂)CT we get:
n₁CT₁ + n₂CT₂ = (n₁ + n₂)CT
Yes?
 
Yes
 
And that rearranges to:
T = (n₁T₁ + n₂T₂)/(n₁ + n₂)
And we know everything on the right hand side so we can calculate the final temperature T.
OK so far?
 
Ohk
 
Then we just use:
P (V₁ + V₂) = (n₁ + n₂)RT
To calculate the final pressure P.
 
Yes
 
8:00 AM
And that's it.
 
Pretty interesting.
Amazing one sir.
I got it too.
 
:-)
 
ohk sir. Thanks a lot. See you soon.
 
Bye :-)
 
Also , I hope you’re safe and healthy. Take care sir.
 
 
1 hour later…
9:13 AM
@JohnRennie Hii sir
1
Q: Doubt regarding proof of $pV = nRT$

Srijan M.TIn my textbook, the proof for this formula is as follows: $$F \propto \text{Area}_{\text{wall}} \cdot \text{(number of density of particles I.e here m is const) I.e} \frac{n}{V} *\text{Average Kinetic energy of particles}$$ Then, we get $$\frac{F}{A} = R(\text{Prop const}) \cdot \frac{n}{v} \c...

Do have a look at this Q
 
It isn't clear to me where the problem is.
Which of the two equations you give do you mean by equation 1?
 
9:29 AM
@JohnRennie Ohk
The top one which is in yellow colour background
>$$F \propto \text{Area}_{\text{wall}} \cdot \text{(number of density of particles I.e here m is const) I.e} \frac{n}{V} *\text{Average Kinetic energy of particles}$$
This
If F is directly proportional, then it means ma is directly proportional
Ok so far ?
 
So you're asking how that equation is derived?
 
There two things I ask kind of
One is derivation
other one is that
Can you prove it that after simplifying the equation or simplifying it dimensionally
It is equal to ma or F
Is it ok? I’ll draw and explain if not clear
 
The equation is correct, but it isn't derived directly. Or at least that isn't how I would derive it.
 
Yes. I want to know how to derive it.
 
Start with a cube of side L containing particles with mass m moving at velocity v.
 
9:35 AM
Ok
 
As a first approximation we'll assume the particles are moving parallel to one of the sides, so they are striking one wall of the cube at normal incidence.
OK so far, or should I draw a diagram?
 
I understood it. Just like 10 person running parallel to each other and in same direction
 
OK, now consider one molecule hitting the face of the cube. It hits the face at velocity v then bounces back with velocity -v so the change in momentum is Δp = 2mv. Yes?
 
Yes
 
Now, we are going to calculate the total momentum change in one second because this is dp/dt and we know F = dp/dt. So if we calculate the momentum change in one second we know the average force the molecule exerts on the face of the cube.
OK so far?
 
9:40 AM
one sec
Yes
Can you explain a little what this average force would mean ? Like force applied in the time interval 0-1 sec
 
OK. When the molecule hits the wall it bounces back. Then it has to travel a distance L across the cube, bounce off the opposite face then travel a distance back to the face again. So in between collisions with the face it has to travel a distance 2L. Yes?
 
Ohk
 
And the time it takes to travel a distance 2L is t = 2L/v, so the time in between collisions is t = 2L/v. Yes?
 
The blue particle I drew
 
Yes
 
9:44 AM
@JohnRennie Ohk
 
So the number of collisions with the face per second is 1/t and that is v/2L.
And each collision changes the momentum by 2mv
 
Does it travel L distance in 1sec
 
So the total change of momentum per second is v/2L x 2mv = mv²/L
 
or 2L in 1sec
?
 
@SrijanM.T suppose we are calculating the force on the right face. Then after the molecule collides with the right face it bounces back and starts travelling left.
 
9:47 AM
Yes
 
It travels a distance L then it hits the left face. When it hits the left face it bounces again and starts travelling right.
 
Yes
 
Then it travels another distance L before it hits the right face again.
So the distance it has to travel between collisions with the right face is 2L.
OK so far?
 
Yes
 
And if it is travelling at a speed v then the time it takes to travel a distance 2L is t = 2L/v
Yes?
 
9:49 AM
Yes
@JohnRennie I didn’t understand 1/t?
 
Suppose you hit something every 0.1 seconds, how many times a second do you hit it?
 
1/10 times
Right ?
 
Really? If you hit something every 0.1 seconds then the number of times you hit it in one second is 1/10?
 
In total 1sec = 10times
 
Yes. Because 10 = 1/0.1
Yes?
 
9:52 AM
Yes. Sorry , didn’t get it quick.
Understood now
 
So if the time in between collisions with the surface is t then the number of hits per second is 1/t.
And 1/t = v/2L.
 
Yes sir
 
And at each collision the momentum change is 2mv, so the total momentum change per second is v/2L x 2mv = mv²/L
 
Yes sir
 
OK. That was for one molecule. But now suppose we have n molecules in the box, then to get the total momentum change for all n molecules we just multiply by n. So we get the total momentum change in 1 second:
Δp = n mv²/L
OK so far?
 
9:56 AM
Yes sir. Right
 
But the momentum change per second is just the force, because F = dp/dt. Yes?
 
Yes
 
And pressure = F/A
 
Yes
Right
 
For our cube the area of the face is L² so we can calculate the pressure by dividing both sides by L². That gives:
P = n mv²/L³
 
9:58 AM
Ok
 
And n/L³ is the number of particles per unit volume i.e. the number density. Let's call this N. And mv² is twice the KE, so we can write our equation as:
P = N x 2KE
 
Hmm K. Now we’re getting it.Nice.understood
 
In your question you have multiplied both sides by the area to get the force F =PA, so you ended up with the equation:
F = A x N x 2KE
 
Ohk.
I feel we have everything now. Just 2KE should be KE
 
So that's how you get that equation. Your equation has a proportional symbol not equality because actually we used a really crude approximation. In a real gas the molecules are travelling in all directions at random.
 
10:03 AM
Ohk.
Also , They took K.E = Temp
 
So our calculation will have some correction factor that is probably complicated to calculate. We won't worry what the correction fact is. Instead we'll just say the force is proportional to A x N x KE.
@SrijanM.T that's because the average KE of an ideal gas molecule is equal to ³⁄₂kT.
Do you know why that is?
 
No sir. Haven’t reached that far. We did thermo a bit and then gas laws.
I don’t know the reason but only knew that first law equation for ideal gas
 
In classical statistical mechanics, the equipartition theorem relates the temperature of a system to its average energies. The equipartition theorem is also known as the law of equipartition, equipartition of energy, or simply equipartition. The original idea of equipartition was that, in thermal equilibrium, energy is shared equally among all of its various forms; for example, the average kinetic energy per degree of freedom in translational motion of a molecule should equal that in rotational motion. The equipartition theorem makes quantitative predictions. Like the virial theorem, it gives the...
 
Ohk. I will check it for sure then,
 
The equipartition theorem states that at a temperature T every degree of freedom of a system gets an energy of ¹⁄₂kT.
 
10:06 AM
Ohk. So , there I guess the 2 of K.E gets cancelled out
 
And a single particle has three degrees of freedom corresponding to motion in the x, y and z directions.
 
Yes.
 
So the energy of a single particle is 3 x ¹⁄₂kT = ³⁄₂kT
And the only energy a single particle has is its KE. Hence the average KE = ³⁄₂kT
 
Ohk.
 
@JohnRennie, my teacher also confirmed that the charge distribution will not be uniform like you said in that case. And the question I was talking about considered infinitely long conductor so charge density will be uniform in there. So that solves my problem :-)
 
10:08 AM
@Satwik cool :-)
 
Interesting. Thank you very much for today sir. I have solved a major chunk of my part today
 
@SrijanM.T :-)
 
 
3 hours later…
1:17 PM
> Two trains A and B of length 400 m each are moving on two parallel tracks with a
uniform speed of 72 km h–1 in the same direction, with A ahead of B. The driver of
B decides to overtake A and accelerates by 1 m s–2. If after 50 s, the guard of B just
brushes past the driver of A, what was the original distance between them ?
Motion relative to train A

Intial velocity of train B=$0$

Initial distance between the train=$x$
$$800+x=\frac12(1)(50)^2\\x=450m$$
What did I do wrong?
 
Displacement in nth second
$$S_n=un+\dfrac12an^2\\S_{n-1}=u(n-1)+\dfrac12a(n-1)^2$$
$$
\bbox[5px,border:2px solid red]
{S_n-S_{n-1}=u+\frac12a(2n-1)}$$
Isn't this dimensionally incorrect?
 
1:56 PM
https://physics.stackexchange.com/questions/128172/the-dimensional-formula-of-distance-travelled-in-nth-second
Take a look at John Rennie's answer.
 
Yes I observed it a few minutes ago :-D
 
06:00 - 15:0015:00 - 17:00

« first day (1516 days earlier)      last day (1093 days later) »